CCHT STUDY BUNDLE PACK SOLUTION 2023 (Questions and Answers )(Verified Answers)

annually
AAMI Chemical Analysis of water used for hemodialysis is tested

Clean the machine surface immediately
A dialysis technician observes a drop of blood on the surface of a patient’s hemodialysis machine. Which of these actions should the dialysis technician take?

About one ounce (30 ml) fluid is on the floor
A dialysis technician observes all of the below on the treatment floor, which one poses an environment risk?

An artery to a vein
A fistula connects

Vitamin B12
A marker used to determine the middle molecule clearance of a dialyzer is

Cations and anions
A mixed- bed DI tank contains

A glomerulus and a tubule system
A nephron is made up of

An infiltration
A patient complains of pain at the venous needle site. The dialysis technician observes there is swelling distal to the venous needle site and that the venous pressure is rising. The dialysis technician should suspect

Dialysis Disequilibrium Syndrome
A patient who is to receive their first dialysis treatment is prone to?

bananas
A patient tells the technician that the dietician reviewed high potassium foods with her. During the conversation she states she routinely eats the following foods. Which one is highest in potassium?

It’s a new Federal regulation
A reason why a dialysis technician is to be certified is

Clotting of extracorporeal circuit
A rise in transmembrane pressure in a low flux dialyzer during hemodialysis is indicative of

Never document something that someone else did
A teammate asks you to document the normal saline he just gave to Mr. Smith. You would:

May rupture and cause the patient to bleed profusely
A true aneurysm can be dangerous because it

Was the predialysis sample drawn from the venous port?
A patient’s laboratory results indicate that the predialysis BUN level was lower than the post dialysis BUN level. When interpreting these findings the technician should seek the answer to which of these questions?

Tachycardia
A rapid pulse of 104 beats/minute would indicate that the patient has:

500
A patient’s dry weight is 154.0 (70kg) and the patient’s pre treatment weight is 158.4. (72kg). For a four- hour treatment, how many mL/ hour must the patient lose to reach dry weight?

200 CFU/mL
AAMI standards for the total microbial count of dialysate shall not exceed:

3-5% of EDW
Acceptable interdialytic weight gains are

Less than two (2) EU/mL
According to AAMI the standard level for endotoxins levels is

4/15/2010
According to the new Conditions for Coverage, all dialysis technicians are to be certified by a national certifying agency or state specific certification by:

Nutritional
Albumin is an important test that tells about the patient’s

Vitamin B12
All of the following are phosphate binders except:

numbness and tingling
All of the following may be signs and symptoms of an infected access except:

The absence of disease producing organisms
Asepsis is defined as

1,385 mL/hr
Based on Mrs Smith’s total amount of fluid to be removed, what is the ultrafiltration rate per hour (mL/hr) for her 4 hour treatment?

Discard the needle
Before a hemodialysis treatment, the technician removes the cap from the fistula needle and accidentally touches the chair with the tip of the needle. Which of these actions should the technician take first?

Hemolysis
Dialysate that is too hot will cause

Weight at which the patient has no edema or fluid in the lungs and BP is normal
Dry weight is defined as the

Disconnection of the bloodline
During a hemodialysis treatment the technician responds to a low venous pressure alarm. The most likely cause for this alarm is:

Blood
Biohazardous waste includes any material exposed to:

Diffusion
Decreasing the patient’s prescribed blood flow rate affects the rate of

Stop the pump
During a hemodialysis treatment, a patient’s venous needle infiltrates. Which of these actions should the dialysis technician take first?

Between 7.0 and 7.4
Dialysate pH should be:

Clamp the venous line
During hemodialysis, the air detector on Ms. Foster’s machine alarms and foam is visible in the venous drop chamber. The immediate action of the dialysis technician should be to:

How have you been since your last dialysis treatment?
During predialysis data collection, it would be best for the technician to ask the patient which of these questions?

Take the patient’s temperature
During the hemodialysis treatment, a patient complains of feeling chilled. In addition to getting the patient a blanket, which of these actions should the technician take?

Notify the equipment technician
During the set up of a hemodialysis machine, the alarm test for conductivity fails. Which of these actions should the dialysis technician take?

Indicators of the adequacy of the renal replacement therapy
ESRD specific tests such as BUN, Creatinine, and URR are

Ultrafiltration
Excess fluid is forced out of the blood and into the dialysate during dialysis by which principle

Percent rejection
Feed conductivity- product conductivity divided by x 100 is the formula calculation of

The ultrafiltration rate
Fluid to be removed divided by the treatment time tells us

Inadequate dialyzer priming
Germicide rebound occurs due to:

Anaphylaxis (severe allergic reaction)
Hives, itching, tightness of the chest and respiratory distress may be signs and symptoms of:

Cardiac arrest
Hyperkalemia can lead to

Hemoglobin
If the extracorporeal circuit clots and it must be discarded, it would be most important to monitor the patient’s serum level of:

Check dialyzer for total cell volume
In reuse, the blood path integrity is tested to

Subcutaneous
Lidocaine is administered

Higher hospitalization and death rates
Low albumin levels in the dialysis patient have been linked with:

Digoxin
Medications that dialyze easily are all except

Psychomotor
Mr. Jones demonstrates how to wash his aces. This is an example of which adult learning principle?

Milk of Magnesia
Mr. Jones suffers from constipation. He goes to the drug store to buy a laxative. Mr Jones should avoid which medication?

Cognitive
Mr. Roberts verbalized how to hand crank his blood pump in the event of an emergency. This is an example of which adult principle?

Notify the nurse of the need for an assessment
Mrs. Smith arrives for dialysis treatment complaining of shortness of breath. Her pre dialysis weight is 4.7 kilograms above the last post treatment weight. The dialysis technician reweighed and verified the weight. Which of the actions should the dialysis technician take next?

5,540 mL
Mrs Smith dry weight is 62 kg. Her pretreatment weight when she came in for treatment on Monday was 67 kg. The priming saline amount is 240 mL, the rinseback amount is 200mL, fluid from medication is 100 mL, and she is not allowed any fluids during treatment. What is the total amount of fluid weight to be removed during her treatment?

Trendelenburg position on left side
Ms Foster complains of shortness of breath. While calling for the nurse the dialysis technician notices there is air in the extracorporeal circuit she/he should place Ms Foster in:

Air
One reason that normal saline is used to rinse and prime the extracorporeal circuit prior to treatment is to remove:

7.0-7.25
Normal body pH is

Time to mature
One disadvantage of a fistula is an increase in

Poor rotation of sites
One cause of aneurysms and pseudoaneurysms is

Blood pump has stopped
The air detector alarms during treatment. If the machine is working properly, the dialysis technician should expect which of these events to have occurred?

Sterilant infusion
Pain at the venous needle site, along with respiratory difficulty and chest pain could be an indicator of:

Hypertension
One sign and symptom of fluid overload is

Hypo-osmole
Solutions that have a higher similarity than body fluids are called?

Numbness and tingling
Symptoms of hyperkalemia (high potassium) may include

Before treatment, after treatment, and after giving heparin
The accuracy of lab results depends on adherence to proper lab draw technique. You must ensure that labs are drawn:

Dialysis Disequilibrium Syndrome
Short frequent hemodialysis treatments in patient’s with high BUN (blood Urea Nitrogen) are performed to prevent:

An infection
Prior cannulation a technician notices the skin over the patient’s fistula is reddened and warm to touch. These symptoms are most likely due to:

Epogen
A medication used to treat anemia is

Teammate
The most important monitor in the dialysis setting is:

Infection
The most frequent cause of death in the first year post transplant is:

12-20 breaths per min
The normal range of resting respirations in an adult is

Diabetes
The leading cause of chronic kidney disease for adults in the United States:

24 hours
The minimum dwell time to achieve sterilization with the chemical Renalin is

Cleansing of the hands
The most important intervention to prevent the spread of infection is

Osmosis
The movement of fluid from the tissue towards a higher concentration of sodium in the blood is an example of

Nephron
The functional unit of the kidney, or the structure in the kidney that does the work, is the

Listening with a stethoscope
The bruit in a Av fistula or graft is detected by

Temperature and transmembrane pressure
The hemodialysis machine continuously monitors many parameters. Which of these parameters must be verified by an external measuring device?

65%
The National standard, based on the KDOQI (kidney Disease Outcomes Quality Initiative) guidelines, for the Urea Reduction Rate (URR) is a ratio greater than:

Peritonitis
The most common complication in Peritoneal dialysis is:

Patient
The most important member of the patient care team is the

Zemplar
The medication used to improve calcium absorption is:

6 months
The minimum amount of experience required to take the NNCC certifying exam is:

1.0 to 1.5 liters per day
The usual fluid allowance for a person with kidney failure is

AAMI
The organization that sets the standards for dialysis water quality is called

Performance of the dialyzer
Total cell volume (TCV) is an indirect measure of the:

Before each patient shift or every four hours
Total chlorine levels in the water are tested

Increase the rate of waste removal
The purpose of using counter-current flow is to

Osmosis
Water moves across a membrane from an area of lower solute concentration to an area of higher solute concentration by which principle

Operates on the principle of ion exchange
The water softener

80%
The TCV should be no less than

Edema and shortness of breath
What are the typical signs of fluid overload

A thrill
To determine the latency of an arteriovenous vascular access, the dialysis technician should initially feel for the presence of

Transmembrane pressure
Ultrafiltration is influenced by

Diffusion
Waste products are removed during dialysis by which principle

Calcium and magnesium
The water softener removes

Residual Sterilant
The reprocessed dialyzer is carefully rinsed prior to patient use to remove

Endotoxin
The part of the cell wall of bacteria that, when released, can cause fever in hemodialysis patient’s is:

Should point in the direction of the blood flow
The venous needle in an AV fistula

Transmembrane pressure
The total pressure that exists across the dialyzer membrane is called

Aluminum
The substance that may be added to the municipal drinking water to make it clearer is:

Kt/V
Which of the following is the most accurate measure of dialysis adequacy?

Bicarbonate
Which of the following component in dialysate is used to correct metabolic acidosis?

Kinking of the venous blood line
Which of the following is most likely to cause a high venous pressure alarm?

Hemoglobin/Hematocrit
What lab tests are used to monitor anemia in the ESRD patient?

Cognitive
Which learning domain is knowledge based?

Cranberry juice colored blood in the venous line
Which of the following is the classic sign of hemolysis?

Ice Cream
Which food below is highest in phosphorus?

O.9% normal saline
When fluid replacement is necessary during the hemodialysis treatment, which of the following solutions is used?

Elevated hematocrit
Which of the following is not a symptom of uremia?

Removing too much fluid
Which of the following is a cause of hypotension during and after dialysis treatments?

Dialysis quality water
Which of the following is considered to be safe water for dialysis?

18 gauge
Which of the following is the largest needle?

Using a bath that will provide a greater concentration gradient
Which of the following actions would increase diffusion during dialysis?

Urea
Which of the following diffuses from the patient’s blood during dialysis?

Handwashing
What is the most important activity that the dialysis technician can perform to prevent the spread of infectious disease in the dialysis clinic?

2.0 inches apart
When cannulating. The tips of the needles should be at least

Hypotension and cramping
Which of the following symptoms would indicate a need to increase the estimated dry weight (EDW)?

2.0
Your patient is to receive 2000 units of heparin. The vial you are using is 1000 u/ml heparin. How many mls will you draw up for this patient?

An arteriovenous fistula
Which of these types of vascular accesses has the fewest complications?

Tighten all connections
Which of the following is used to prevent air in the bloodlines?

AAMI
Which organization set the standards and recommendations for dialyzer reprocessing?

Venous needle is against the wall of the vessel
Which of the following would cause a high venous pressure alarm?

Clotting in the extracorporeal circuit
Which of the following might indicate a need to increase heparin?

The patient just had surgery yesterday
Which of the following might indicate a need to decrease the heparin dose?

Ensuring accurate documentation
Which of these technician activities contributes to data collection used to measure quality outcomes?

Draw one line through it, date, time, signature/credentials, with the words “error”
You have documented incorrectly on a patient’s chart. How do you correct this?

Phosphorus, potassium, and calcium
What’s the normal lab value?

The movement of particles across a semipermeable membrane from an area of higher solute concentration to an area of lower solute concentration, until the concentration on both sides of the membrane is equal
What is diffusion?

The process of passing fluid through a filter or semipermeable membrane
What is filtration?

A medication(synthetic erythropoietin) used to stimulate production of red blood cells
What is epogen?

2 hours
How long can a used dialyzer sit out before reprocessing?

11 hours
How long can a reprocessed dialyzer sit out to dry before use?

14 days
What is a reused dialyzer shelf life?

80%
What is a reuse dialyzer (TCV) percentage

Association for the Advancement of Medical Instrumentation
What does AAMI stand for?

Bicarbonate
The base used to maintain an acceptable dialysate pH(acid/base balance). A buffer

Blood urea nitrogen, the end product of protein metabolism
What is BUN?

A ballooning of the AV fistula, sometimes caused by improper rotation of needle sites
What is a aneurysm?

dysrhythmia
An irregular heart beat

Dyspnea
Difficulty breathing

Edema
Abnormal accumulation of fluid in the body tissues causing swelling. Fluid accumulated in the interstitial space

A medication (synthetic erythropoietin) used to stimulate production of red blood cells
What is Epogen given for?

The circuit “outside body” that carries blood from the patient through dialyzer and back to the patient. The dialyzer and blood lines make up the circuit
What is extracorporeal circuit?

The process of passing fluid through a filter or semipermeable membrane.
What is filtration?

A tuft of capillaries in the nephron that act as filters to remove waste products, fluid and excess electrolytes from the blood as it passes through the kidney.
What is glomerulus?

Hematocrit (HCT)
The measurement of the percentage of red blood cells in a sample of whole blood. This measure is affected by vascular volume.

Hematoma
A localized mass of clotted or partially clotted blood confined within an organ or tissue, which may be discolored (bruised).

Hemoglobin (HB)
A protein that carries oxygen from the lung to the body tissue and transports carbon dioxide from the tissues to the lungs.

Hemolysis
What is the rupture of red blood cells? ( lysis is cell rupture)

Hemostasis
The cessation of bleeding due to clotting. In dialysis, hemostasis refers to the cessation of bleeding from a needle site.

hyperkalemia
An excess of potassium in the blood caused by dietary indiscretion, certain medications, internal bleeding, cellular injury or blood transfusion.

Muscle weakness, nausea, and vomiting
What are symptoms of hyperkalemia?

Hypertension
High blood pressure is signs of?

Movement of blood from a needle site into the surrounding tissue that occurs when the needle pierces the vessel wall.
What is infiltration?

Swelling around the insertion site and pain
Symptoms of infiltration?

A mathematical measurement of the adequacy or dialysis treatment, which considers protein intake and the amount of urea removed from the patient.
What is kt/v?

Nephron
The functional unit of the kidney

Osmosis
The movement of fluid across a semipermeable membrane from an area of lower solute concentration to an area of higher solute concentration. Movement occurs until equilibrium is reached:

pH
The measure of the acid-base status of a solution. PH stands for potential of hydrogen

An electrolyte necessary for bone formation, energy transfer, andneuromuscular function
What is phosphorus?

Very thin flexible sheets of material, such as cellophane, with pores that allow smaller particles or molecules to pass through while blocking larger ones.
What is semipermeable membrane?

A condition caused when the surgical diversion of the arterial blood supply through the vascular access deprives the distal portion of the access extremity of oxygenated blood.
What is steal syndrome?

A narrowing of blood vessels due to the build up of hypertrophic tissue, anatomical narrowing if the vessels, or surgical technique problems.
What is stenosis?

Rapid heart beat
What is Tachycardia?

Controlled fluid removal achieved by increasing or decreasing the amount of fluid moves across a semipermeable membrane by controlling hydrostatic pressure
What is ultrafiltration?

A waste product of protein metabolism affected by dietary intake.
What is urea?

A vitamin that helps the body produce normal size cells with strong cell walls.
What is vitamin B-12?

A vitamin that, in its active form helps maintain the balance of blood calcium and phosphorus with calcium and phosphorus stored in the bones.
What is vitamin D?

A slow heart rate
What is bradycardia?

Hypertension
One sign and symptom of fluid overload?

Osmosis
Water moves across a membrane from an area of lower solute concentration to an area of higher solute concentration by which principle?

Diffusion
Waste products are removed during dialysis by which principle?

Ultrafiltration
Excess fluid is forced out of the blood and into the dialysate during dialysis by which principle?

Osmosis
The movement of fluid from the tissue towards higher concentration of sodium in the blood is an example of

Diffusion
Decreasing the patient’s prescribed blood flow rate affects the rate of?

Transmembrane pressure
Ultrafiltration is influenced by

11 hours
The minimum dwell time to achieve sterilization with the chemical renalin is?

Residual Sterilant
The reprocessed dialyzer is carefully rinsed prior to patient use to remove:

Box port behind the machine
The patient’s machine alarms blood leak, where would you draw the sample from to determine if there is a blood leak?

Pyrogen Reaction
What causes symptoms of fever and chills?

arteriovenous (AV) fistula
Which of the following types of vascular access has the lowest incidence of infection?

Hypotention
During Mrs. King’s hemodialysis treatment, she starts actively yawning. The technician suspects she may be experiencing

Salt level in brine tank
While performing the end-of-day water hardness check, the technician notes that the hardness exceeds the allowable limit. In addition to notifying the equipment technician, the technician should check which component of the water system?

A life-threatening condition
Chloramine levels in treated water that are in excess of the standards of the Association for the Advancement of Medical Instrumentation (AAMI) could result in

Salt
A patient is experiencing large weight gains between hemodialysis treatments. The technician should remind the patient that the best dietary measure to control weight gain is to decrease the intake of

Notify the equipment technician
When doing a routine patient check, the technician notices a puddle of clear liquid underneath a dialysis machine. After wiping it up, which of these actions, if any, should the technician take?

Rope and ladder technique
A patient has an arteriovenous (AV) fistula in his lower left arm. When choosing a cannulation site, the technician observes where previous needles were placed and chooses to use alternate sites for arterial and venous needle insertion. This practice is an example of

Hypertonic
A solution with higher osmolarity than body fluid?

Call the bio-med
If during a set up, a machine test fails what would be the next step after re-testing?

Operates in the principle of ion exchange
What does the water softener do?

Dialysis Equilibrium Syndrome
Shortened frequent treatments in patient’s with high BUN prevent?

1:100
What is the ratio for bleach wipes?

1:10
What is the ratio for bleach clamps?

1:10
What ratio do you use for blood spills?

1:10
What is the ratio for bleach water?

Numbness and tingling
Symptoms of hyperkalemia?

Endotoxins
What part of the cell wall when released can cause fever in hemodialysis patients?

Hemoglobin
If the extracorporeal circuit clots we must monitor the patient’s serum level of?

Using a bath with a greater concentration
Which action increases diffusion during dialysis?

Residual Sterilant
Dialyzers are carefully rinsed prior to use to remove:

Hecterol
Vitamin given to patients by the nurse?

Blood electrolyte levels
Changing the composition of the dialysis solution can influence the patient’s

Remove the line from the venous line clamp
Blood flow is manually maintained during power outage. The technician must take which of the following actions?

Countercurrent flow
The technician observes that a patient’s dialysate lines were incorrectly placed on the dialyzer. The technician corrects the line placement. The principle underlying the technicians’ actions is that the clearance is maximized by

Treatment time
Mrs. Miller’s calculated kt/V urea is 0.9 one of the factors that affects her kt/V urea is her?

As far apart as possible
In order to maximize access recirculation. The technician should place Mrs. Miller’s fistula needle:

Clot formation
Heparin is given during Mrs. Miller’s dialysis treatment to lower risk for

We need to ensure early detection of needle dislodgement
Mrs. Miller asks the technician”why can’t I completely cover up with a blanket during my hemodialysis”? “I’m cold” which of these responses by the technician would be best?

Make sure the blood pump stopped
During a treatment, Mrs. Miller’s venous needle infiltrates. Which of the actions should the technician take first?

It is not permitted by the current ESRD regulations
A patient is busy with a patient when a machine across the room alarms. The dietician standing nearby mutes the alarm. The technician should have which of these understandings concerning this action?

Steal
A patient who recently had an Av fistula placed in the left forearm is experiencing numbness and tingling of the left hand. The patient’s left is cold and fingernail beds are blue. The patient most likely had which of of these symptoms?

Thrombosis
Holding needle puncture sites instead of using spring loaded clamps fistula clamps is recommended to avoid access

Disinfecting
The greatest reduction in microorganism in the dialysis machine is accomplished by

Teaching patients how to meet nutrition needs
The role of the renal dietician includes

Hypotension
During Mrs. King’s hemodialysis treatment, she starts actively yawning. The technician suspects she may be experiencing

First- use syndrome
A patient who is receiving an initial hemodialysis treatment on a non- reusable dialyzer should be observed for signs of

Creatinine
A substance that is cleared from the body by healthy kidney is?

Whether other patients are having similar symptoms
During a dialysis treatment, a patient suddenly develops fever and chills. The initial action that should be taken is to determine

Air detector
Which of the following components of the dialysis machine is designed to protect the patient from bubble/foam?

Remove disinfectant
One purpose of priming the bloodlines and extracorporeal circuit pre-dialysis is to?

Adequately rinseback as much of the patient’s blood as possible at each treatment
Which of the following actions can the dialysis technician take to help minimize anemia?

7 days
The hepatitis B virus is able to survive on environmental surfaces for as long as:

Change the DNA of microbes so they die or cannot multiply
A new technician is being oriented to the water treatment system. The equipment technician explains that the purpose of the ultraviolet (UV) light is to?

Falls
For pain management, Mr Rivera is taking narcotic pills as prescribed by his doctor. The dialysis technician knows that taking this medication puts Mr Rivera at increases risk for

Time required to mature
What is a disadvantage of an AV fistula as hemodialysis access compared to other types of access?

Prevent prolonged bleeding
A patient who has an arteriovenous fistula receives a bonus dose of heparin at the beginning of a hemodialysis treatment and a continuous heparin infusion during the treatment. The patient’s heparin administration should be terminated 30-60 minutes before the end of treatment in order to

Vascular surgeon
A patient with a right Av fistula arrives at the dialysis facility complaining of pain, tingling, and cold fingers in his right hand. The technician would expect the nurse will notify which of the following physicians for possible corrective action?

No additional precautions are necessary for the patient
A patient is diagnosed with hepatitis C beyond standard precautions. What additional measures. If any, should be taken for the patient?

You don’t have enough red blood cells to carry oxygen
A patient tells the technician. “The doctor told me I’m anemic what does that mean”? Which response by the technician would be correct?

Two red blood cells
In hemodialysis, the term anastomosis refers to the connection of?

Blood returning to the patient
Venous pressure is a result of resistance created by the

Before each patient shift
According to the standards of the Association for the Advancement of Medical Instrumentation (AAMI), pre treatment water should be checked for chloramines at which of these times?

The patient must receive dialysis in a hospital in a special isolation room
A patient on hemodialysis goes to the emergency department for persistent coughing and fever. He is diagnosed with active tuberculosis (TB) which of the following actions would the technician expect to be taken in regard to the patient’s dialysis treatment?

Anaphylaxis (severe allergic reaction)
Hives, itching, tightness of the chest and respiratory distress may be signs and symptoms of:

Ultrafiltration
Excess fluid is forced out of the blood and into the dialysate during dialysis by which principle

6-8 weeks
How long does it take a Fistula to mature

1.Which of the following
best describes the reason for wearing gloves when handling machine prior to blood?
a) patient comfort
b) asepsis
c) universal precautions
d) HIV
c) universal precautions

2 You test for sterilant levels in a
hemodialysis machine prior to patient use. The test
is positive for sterilant. This means the machine:
a) is sterile & bacteria free
b) needs further rinsing
c) needs more sterilant
d) is in conductivity
b) needs further rinsing

3.Choose the correct
statement regarding
serum abnormalities in
renal failure:
a) creatinine, BUN, &
phosphorus are elevated
b) creatinine BUN creatinine, BUN, &
phosphorus are depressed
c) calcium & bicarbonate are
elevated
d) calcium & bicarbonate are
depressed
a) creatinine, BUN, &
phosphorus are elevated

4.Hemodialysis requires
a means to channel the
patient’s blood to the
hemodialyzer & then back to the body. It’s
called:
a) vascular access
b) hemodialyzer
c) dialyzing fluid delivery system
d) infusion pump
a) vascular access

5.Which of the following is correct regarding a fistula?
a) usually created in the leg, near the ankle
b) usually created in the
forearm, near the wrist
c) are made of salastic tubing
d) none of the above
b) usually created in the
forearm, near the wrist

6.Blood leak detectors are extremely sensitive
& can determine:
a) blood type
b) amount of blood loss
c) blood leakage immediately
d) volume changes
e) serum potassium levels
c) blood leakage immediately

7.Which of the following foods are high in
potassium?
a) milk, ice cream, yogurt
b) legumes, cheese, pizza
c) chips, colas, canned meat
d) bananas, tomatoes, oranges
d) bananas, tomatoes, oranges

8.What lab values are monitored for
assessing bone disease?
a) calcium, phosphorus &
PTH
b) calcium, phosphorus &
albumin
c) calcium, phosphorus &
potassium
d) calcium, phosphorus &
magnesium
a) calcium, phosphorus &
PTH

9.Which is the most common cause of chronic kidney failure in the US?
a) diabetes
b) hypertension
c) polycystic kidney smaller molecular
disease
d) glomerulonephritis
a) diabetes

10.The reason potassium dialyzes and
RBC’s do not is:
a) potassium has a larger molecular weight
b) potassium has a smaller molecular weight
c) RBC’s are not water soluble
d) RBC’s have a negative charge
b) potassium has a smaller molecular weight

11.Acceptable limits for total chlorines in water for hemodialysis are?
a) less than 0 .1mg/L (ppm)
b) 0.5 mg/L to 1 mg/L(ppm)
c) 3.5 mg/L to 5.5 mg/L to 5.5 mg/L(ppm)
d) greater than 6.0 mg/L(ppm)
a) less than 0 .1mg/L (ppm)

12.If a new patient starts with a very high
BUN, dialysis is purposely made less efficient to prevent:
a) rapid decrease in hct
b) dialysis disequilibrium syndrome
c) cardiac arrhythmia’s
d) excessive coagulation
b) dialysis disequilibrium syndrome

13.The following are factors in poor clearance during
dialysis except:
a) poor dialyzer reuse
b) access recirculation
c) low hematocrit
d) low blood flow rate
c) low hematocrit

14.The movement of water from an area of low solute
concentration to an area of high solute
concentration is called:
a) diffusion
b) osmosis
c) ultrafiltration
d) dialysis
b) osmosis

15.Potting compound, casing, fibers and headers are part of a:
a) dialysis machine
b) water treatment system
c) R. O. system
d) dialyzer
d) dialyzer

16.Mr. Jones has a dry weight of 72 kg. He weighed 73 kg after last treatment. Today he weighs 75.5 kg. Assume prime and rinse back =500 ml total. What is Mr.Jones’ fluid removal goal for today?
a) 2.5 liters or 2500 ml
b) 3.0 liters or 3000 ml
c) 4.0 liters or 4000 ml
d) 4.5 liters or 4500
c) 4.0 liters or 4000 ml

17.A hematocrit test indicates which of the
following?
a) concentration of platelets
b) concentration of creatinine in the blood
c) concentration of red blood cells in the blood
d) concentration of white blood cells in the blood
c) concentration of red blood cells in the blood

18.What factors should be considered when
establishing a patient’s dry weight?
a) blood pressure
b) patient well being
c) evidence of dehydration or fluid overload
d) all of the above
d) all of the above

19.In a dialysis machine, the
proportioning system does what?
a) prepare the dialysate to the proper pH
b) warm the dialysate to the proper temperature
c) appropriately mix the dialysate concentrates
with water
d) to monitor the dialysate flow rate
c) appropriately mix the dialysate concentrates
with water

20.The complication in which blood has a
“cherry pop”appearance is:
a) blood leak
b) air embolism
c) hemolysis
d) none of the above
c) hemolysis

21.The use of high a sodium dialysate may
predispose a patient to:
a) fluid overload
b) hypertension
c) thirst
d) all of the above
d) all of the above

22.The capability of a dialyzer to remove fluid, expressed as ml/mmHg/hr is:
a) UF Coefficient
b) clearance
c) surface area
d) priming volume
a) UF Coefficient

23.A pre-pump arterial pressure reading is
measuring:
a) the pressure required to pump the blood
through the dialyzer
b) resistance to blood flow out of the access
c) pressure within the dialyzer
d) none of the above
b) resistance to blood flow out of the access
through the dialyzer

24.What determines the surface area of a
hollow fiber dialyzer?
a) number of fibers
b) internal diameter of fibers
c) length of fibers
d) all of the above
d) all of the above

25.The purpose of using counter-current flow is to:
a) increase the rate of fluid removal
b) increase the rate of waste removal
c) decrease the amount of dialysate used
d) decrease the surface area of the membrane
b) increase the rate of waste removal

26.Chloramine exposure in dialysis can result in:
a) hypernatremia
b) hemolysis
c) pericarditis
d) bleeding
b) hemolysis

27.Part of the reuse process involves the reuse machine testing the dialyzer to verify:
a) the dialyzer is free of sterilant
b) the dialyzer is free of bacteria
c) the dialyzer is free of endotoxins
d) the dialyzer fibers are free of leaks
d) the dialyzer fibers are free of leaks

28.According to AAMI RD52, what is the
maximum allowable level of bacteria in dialysate?
a) 1,000 CFU’s
b) 50 CFU’s
c) 200 CFU’s
d) 500 CFU’s
c) 200 CFU’s

29.According to AAMI RD52, how often
should bacterial cultures be sampled?
a) weekly
b) monthly
c) quarterly
d) annually
b) monthly

30.According to AAMI RD52, what is the action level for bacteria in dialysate?
a) 50 CFU’s
b) 75 CFU’s
c) 100 CFU’s
d) 200 CFU’s
a) 50 CFU’s

31.Bacterial exposure from water may cause which of the following symptoms?
a) hemolysis
b) fever/chills
c) bone disease
d) hypotension
b) fever/chills

32.Who developed the first permanent blood access, the “Shunt”?
a) Scribner and Quinton
b) Kolff
c) Brescia and Cimino
d) Turner
a) Scribner and Quinton

33.Venous pressure meters are calibrated in:
a) PSI
b) mm/Hg
c) mL/hour
d) mg%
b) mm/Hg

34.TPM is measured as:
a) mL/hr
b) mm/Hg
c) mL/min
d) none of the above
b) mm/Hg

35.The hemodialysis machine continuously monitors many parameters. Which of these parameters must be verified by an external measuring
device?
a) blood flow
b) dialysate flow
c) pH and conductivity
d) temperature and transmembrane pressure
c) pH and conductivity

36.Prior to each patient shift, it would be essential to
perform a water quality check for which of these substances?
a) calcium and magnesium
b) chlorine and chloramine
c) organic contaminants
d) inorganic contaminants
b) chlorine and chloramine

37.The term pH is a measurement of the
concentration of:
a) all ions in solution
b) hydrogen ions
c) bicarbonate ions
d) acetate ions
b) hydrogen ions

38.The technician’s role in patient teaching
should be to:
a) assess the patient”s barriers to learning
b) check the patient’s readiness to learn new
information
c) determine the patient’s learning needs
d) Reinforce teaching that the patient has received
from other team members
c) determine the patient’s learning needs

39.What can dialysis staff do to reduce the
risk of being infected with a blood borne pathogen?
a) proper hand washing
b) universal precautions
c) proper sharps disposal
d) all of the above
d) all of the above

40.Which of the following are ways to
handle chemicals safely?
a) wash hands before eating or drinking
b) wear protective clothing
c) don’t eat or drink in the work area
d) all of the above
d) all of the above

41.What is the correct angle in which to cannulate a fistula?
a) 10-15 degrees
b) 25-35 degrees
c) 35-45 degrees
d) it dosen’t matter
b) 25-35 degrees

42.Which of the following affects access recirculation?
a) distance between the tips of the needles
b) presence of stenosis
c) direction of needles
d) all of the above
d) all of the above

43.The technician observes that the hardness reading after the water softener isabove the acceptable
limit. The technician should check the softener tank’s level of:
a) salt
b) chlorine
c) carbon
d) sand
a) salt

  1. A technician observes all of the following in
    the hemodialysis unit. Which one poses an environmental risk?
    a) the wheels of the hemodialysis machines are locked
    b) charts are lying on bedside tables
    c) about one ounce (30 ml) of fluid is on the floor
    d) a patient’s family member is present during treatment
    c) about one ounce (30 ml) of fluid is on the floor
  2. When preparing a bleach solution for cleaning
    equipment, the technician accidentally spills undiluted
    bleach on the floor. Which of these resources will provide the technician with information about how to
    properly handle spill of the chemical?
    a) Material Safety Data Sheet (MSDS)
    b) Centers for Disease Control & Prevention (CDC)
    c) Association for the Advancement of Medical Instrumentation (AAMI)
    d) State Department of Health (DOH)
    a) Material Safety Data Sheet (MSDS)
  3. A conductivity alarm sounds during a
    hemodialysis treatment. The technician should expect which of these events to have occurred?
    a) the blood pump has stopped
    b) the dialysate flow rate has increased
    c) the electrical power source was interrupted
    d) the dialyzer was bypassed
    d) the dialyzer was bypassed
  4. During routine water testing, the technician notes
    that the chloramine level after the first carbon tank is 0.2mg/L. Which of these actions should the technician
    take next?
    a) bypass the carbon tanks
    b) descale the reverse osmosis membranes
    c) obtain a sample after the second
    d) immediately terminate hemodialysis treatments
    c) obtain a sample after the second
  5. A female technician has been caring for a male patient for several months. The patient tells the technician that he is lonely and depressed. He asks the
    technician to go out to dinner with him. In addition to politely declining the patient’s invitation, which of these actions, if any, should be taken:
    a) refer the patient to the social worker
    b) recommend an over-the-counter mood stimulant, such as St. John’s wort
    c) share the patient’s need for socialization with other patients
    d) no further action is needed
    a) refer the patient to the social worker
  6. A hemodialysis technician suspects that a patient who is receiving a hemodialysistreatment has an air embolus. After stopping the blood pump, the patient
    should be placed in which of these positions?
    a) low-fowlers
    b) high-fowlers
    c) right-sided trendelenberg
    d) left-sided trendelenberg
    d) left-sided trendelenberg
  7. During hemodialysis a patient goes into
    cardiac arrest. Which of these actions shouldbe taken first?
    a) apply a precordial thump to the patient
    b) call for help
    c) return the patient’s blood
    d) stop ultrafiltration
    b) call for help

the function unit of the kidney, or the structure in the kidney that does the work, is the
nephron

the leading cause of chronic kidney disease for adults in the united states:
diabetes

a nephron is made up
a glomerulus and a tubule system

which of the following is not a symptom of uremia
elevated hematocrit

Mrs. smith’s dry weight is 62 kg. her pretreatment weight when she came in for treatment on monday was 67kg. the priming saline amount is 240ml, fluid from medication is 100 ml , and she is not allowed any fluids during treatment. what is the total amount of fluid weight to be removed during her treatment?
5,540ml

based on Mrs. smith’s total amount of fluids to be removed, what is the ultrafiltration rate per hour (ml/hr)for her 4-hour treatment?
1,385 ml/hr

which of the following is a cause of hypotension during and after dialysis treatments?
removing too much fluids

the scale says a patient weights 70 kg. this the same as weighting how many points (lb)?
154

normal body ph is
7.35-7.45

one sigh and symptoms of fluids overload is
hypertension

abnormal levels of this electrolyte can cause cardiac arrhythmias and even death
potassium

water moves across a membrane from an area of lower solute concentration to an area of higher solute concentration by which principle
osmosis

waste products are removed during dialysis by which principle
diffusion

excess fluid is forced out of the blood and into the dialysis by which principle
ultrafiltration

the movement of the fluid from the tissue toward a higher concentration of sodium in the blood is a example of
osmosis

decreasing the patient’s prescribed blood flow affects the rate of
diffusion

which of the following actions would increase diffusion during dialysis?
using a bath that will provide a grater concentration gradient

which of the following diffusion diffuses from the patients blood during dialysis?
urea

ultrafiltration is influenced by
transmembrane pressure

the most common complication in peritoneal dialysis is
peritonitis

the dextrose used in the dialysate for peritoneal dialysis can cause
hyperglycemia

if a patient develops an infection post transplant, the largest contributing factor would most likely be:
immunosuppressive medication

the most frequent cause of death in the first year post transplant is
infection

total cell volume (TCV) is a indirect measure of the
performance of the dialyzer

the TCV should be no less than
80%

the manimum dwell time to achieve sterilization with the chemical renalin is
11 hours

the reprocessed dialyzer is carefully rinsed prior to patient use to remove
residual sterilant

germicide rebound occurs due to
inadequate dialyzer priming

which organization set the standards and recommendations for dialyzer reprocessing?
AAMI

the blood path integrity is tested to eliminate the possibility of a blood leak. the drop in pressure should not exceed:
10 mmhg

the purpose of using counter-current flow is to
increase the rate of waste removal

the total pressure that exists across the dialyzer membrane is called
transmembrane pressure

dialyser ph should be
between 7.0 and 7.4

dialyzer that is too hot will cause
hemolysis

which of the following component in dialysate is used to correct metabolic acidosis?
bicarbonate

total chlorine levels in the water are tested
before each patient shift or every four hour

the water softener removes
calcium and magnesium

AAMI standars for the total microbial count of dialysate shall not exceed
200 CFU/ML

the water softener
operates on the principle of ion exchange

which of the following is considered to be safe water for dialysis
dialysis quality water

patient expose to chloramines during the hemodialysis treatment may cause
hemolysis

according to AAMI the standard levels for endotoxin levels is
less than two (2) EU/ML.

feed conductivity- product conductivity divided by x 100 is the formula calculation of
percent rejection

a mixed-bed DI tank contains
cations and anions

the substance that may be added to the municipal drinking water to make it clearer is
aluminum

AAMI chemical analysis of water used for hemodialysis is tested
annually

the part of the cell wall of bacteria that, when released, can cause fever in hemodialysis patients is
endotoxin

during a hemodialysis treatment the technician responds to a low venous pressure alarm. the most likely cause for this alarm
disconnection of the blood line

the hemodialysis machine continuously monitors may parameters. which of the these parameters must be verified by an external measuring device?
ph and conductivity

which of the following is most likely to cause a high venous pressure alarm?
kinking of the venous blood line

during the set-up of a hemodialysis machine, the alarm test for conductivity fails. which of these actions should the dialysis technician take?
notify the equipment technician

rinse in transmembrane pressure in a low flux dialyzer during hemodialysis is indicative of
clotting of the extrecorporeal circuit

one reason that normal saline is used to rinse and prime the extracorporeal circuit prior to treatment is to remove
air

which of the following best describes the transducer protector?
device inside the machine that converts air pressure into electronic signal

which of the following would cause a high venous pressure alarm?
venous needle is against the wall of the vessels

the air detector alarms during treatment. if the machine is working properly, the dialysis technician should expect which of these events to have occurred
blood pump has stopped

the extracorporeal circuit consists of the
blood lines and the dialyzer

before a hemodialysis treatment , treatment , the technician removes the cap from the fistula needle and accidentally touches the chair with the tip of the needle. which of these actions should the technician take first
discard the needle

to determine the potency of a arteriovenous vascular access, the dialysis technician should initially feel for the presence of
a thrill

a patient complaints of pain at the venous needle site. the dialysis technician observes there is swelling distal to the venous needle site and that the venous pressure is rising. the dialysis technician should suspect
an infiltration

all the following may be signs and symptoms of an infection access EXCEPT:
numbness and tingling

during a hemodialysis treatment, a patient’s venous needle infiltrates. which of these actions should the dialysis technician take first?
stop the blood pump

which of these types of vascular access has the fewest complication
an arteriovenous fistula

one disadvantage of a fistula is an increase in
in time to mature

one cause of aneurysm and pseudo aneurysm is
poor rotation of sites

what is the desired outcome when the patient performs fistula exercise?
enlargement of the vein

the bruit in an AV fistula or graft is detected by
listening with a stethoscope

when cannulating, the tips of the needles should be at least
2.0 inches apart

the venous needle in an AV fistula
should point in the direction of the blood flow

a true aneurysm can be dangerous because it
may rupture and cause the patient to bleed profusely

a fistula connects
an artery to a vein

prior to cannulation a technician notices the skin over the patient’s fistula is reddened and warm to touch. these symptoms are most likely due to
infection

during predialysis data collection, it would be best for the technician to ask the patient which of these questions?
how have you been since your last dialysis treatment?

Mrs. pruitt’s temperature is elevated post-hemodialysis treatment. which of these actions should the dialysis technician take?
notify the nurse

Mrs. smith arrives for dialysis treatment complaining of shortness of breath. her pre-dialysis weight is 4.7 kilograms above the last post-treatment weight. the dialysis technician reweighed and verified the weight. which of the actions should the dialysis technician take next?
notify the nurse of the need for an assessment

slow pulse of 54 beats/minute would indicate the patient has:
bradycardia

during hemodialysis, the air detector on Ms. foster’s machine alarms and foam is visible in the venous drip chamber. the immediate action of the dialysis technician should be to.
clamp the venous line

dry weight is defined as the
weight at which the patient has no edema or fluids in the lungs and bp is normal

which of the following symptoms would indicate a need to increase the estimated dry weight (EDW)?
hypotension and cramping

Sam’s EDW is 70 kilograms (KG). he arrives for treatments today at a weight of 70kg. he complains of shortness of breath and you notice his ankles are swollen. you notify the charge nurse. what do you think will be the plan for sam’s treatments todays?
the nurse will get an order to decrease the EDW

which of the following is not a possible cause of an increased temperature pre-dialysis?
the patient has fluid overload

which of the following is the classic sign of hemolysis
cranberry juice colored blood in the venous line

Ms. foster complication of shortness of breath. while calling for the nurse the dialysis technician notices there is air in the extracorporeal circuit she/he should place Ms. foster in:
trendelenburg position on left side

solution that have a higher osmolarity than body fluids are called?
hypertonic

the most important intervention to prevent the spread of infection is:
cleansing of the hands

infection control protocol for hemolysis equipment should include which of these actions by the caregiver?
using gloves when touching the machine

when fluid replacement is necessary during the hemodialysis treatment, which of the following solution is used?
0.9% normal saline

a patient’s dry weigh is 154.0 lb (70 kg) and the patient’s pre treatment weight is 158.4 lb (72kg). four-hours treatment, how many ml/hour must the patient lose to reach dry weight?
500

during a hemodialysis treatment, a patient says to the technician, three hours of time have passed according to the clock on the wall, but the machine shows that only 2.5 hours have passed. what could be the problem? the technician should understand that the.
machine was in bypass for 30 minutes

a dialysis technician observes a drop of blood on the surface of a patient’s hemolysis machine. which of these actions should the dialysis technician take?
clean the machine surface immediately

asepsis is defined as
the absence of disease producing organism

if the extracorporeal circuit clots and it must be discarded, it would be most important to monitor the patient’s serum levels of
hemoglobin

what are the typical signs of fluids overload
edema and shortness of breath

hyperkelemia can lead to
cardiac arrest

which of the following is used to prevent air in the bloodlines?
tighten all connections

fifteen minutes before the end of a hemodialysis treatment, a patient complains of dizziness and nausea. these symptoms are most likely due to
hypotension

during the hemodialysis treatment, a patient complains of feeling chilled. in addition to getting the blanket, which of these actions should the technician take?
take the patient’s temperature

short frequent hemodialysis treatment in patients with high BUN ( blood urea nitrogen ) are performed to prevent
dialysis disequilibrium syndrome

during hemolysis a patient goes into cardiac arrest. the first action of the dialysis technician should be?
call for assistance

a patient who is to receive their first dialysis treatment is prone to
dialysis disequilibrium syndrome

fluid to be removed divided by the treatment time tells us
the ultrafiltration rate

the normal range of resting respirations in an adult is:
12-20 breaths per minute

hives, itching, tightness of the chest and respiratory distress may be signs and symptoms of:
anaphylaxis (severe allergic reaction)

pain at the venous needle site, along with respiratory difficulty and chest pain could be an indicator of:
sterilant infusion

a rapid pulse of 104 beats/minute would indicate that the patient has:
tachycardia

a marker used to determinate the middle molecule clearance of a dialyzer is
vitamin B12

the national standard, based on the KDOQI ( kidney disease outcomes quality initiative ) guidelines, for the urea reduction rate ( URR) is a ratio greater than :
65%

what lab tests are used to monitor anemia in the ESRD patient?
hemoglobin/hematocrit

ESRD specific tests such us as BUN, creatinine, and URR are
indicators of the adequacy of the renal replacement therapy

the accuracy of lab results depends on adherence to proper lab draw technique.you must ensure that labs are drawn:
at the right time, right site, and right order

albumin is an important test that tells about the patient————status
nutritional

which of the following is the most accurate measure of dialysis adequacy?
Kt/V

biohazardous waste includes any material exposed to
blood

a patient laboratory results indicate that the predialysis BUN levels was lower than the post dialysis BUN levels. when interpreting these findings the technician should seek the answer to which of these questions
was the predialysis sample draw from the venous port?

the organization that has as a goal to ” improve the care and outcomes of all individuals with kidney disease” is called
KDOQI

a dialysis technician observes all of the below on the treatment floor, which one poses an environment risk?
about one ounce (30ml) fluid is on the floor

what is the most important activity that the dialysis technician can perform to prevent the spread of infectious disease in the dialysis clinic?
hand washing

the organization that sets the standards for dialysis water quality is called
AAMI

which of these technician activities contributes to data collection used to measure quality outcomes?
ensuring accurate documentation

a patient tells the technician that the dietitian reviewed high-potassium foods with her. during the conversation she states she routinely eats the following food. which one is highest in potassium?
bananas

which food below is highest in phosphorus
ice cream

low albumin levels in the dialysis patient have been linked with:
highest hospitalization and death rates

acceptable interdialytic weight gains are
3-5% of EDW

the usual fluid allowance for a person with kidney failure is
1.0 to 1.5 liters per day

symptoms of hyperkalemia (high potassium) may include
numbness and tingling

which of the following might indicate a need to decrease the heparin dose?
the patient just had surgery yesterday

which of the following might indicate the need to increase heparin
clotting in the extracorporeal circuit

the usual heparin sodium concentration for anticoagulation during hemolysis is
1,000u/ml

lidocaine is administered
intradermally

medications that dialyze easily are all expect:
digoxin

Mr. jones suffers from constipation. he goes to the drug store to buy a laxative. Mr. jones should avoid which medication?
milk of magnesia

which of the following is the largest needle?
18 gauge

your patient is to receive 2000 units of heparin. the vial you are using is 1000u/ml heparin. how many mis will you draw up for this patient?
1.0

all of the following are phosphate binders expect:
vitamin B 12

the medication used to treat anemia is
epogen

the medication used to improve calcium absorption is:
zemplar

a female dialysis technician has been caring for a male patient for several months. the patient tells the dialysis technician that he is lonely and depressed. he asked the dialysis technician to go out to dinner with him. in addition to politely declining the patient’s invitation, which of these actions, if any, should the dialysis technician take?
refers the patient to the social worker

you have documented incorrectly on a patient’s chart. how do you correct this?
draw one line through it, date, time, signature/credentials, with the works ” error”

a teammate asks you to document the normal saline he just gave to Mr.smith you would
never document something that someone else did

Mr. jones demonstrates how to wash his access. this is a an example of which adult learning principle
psychomotor

the dialysis technician’s role in patient education is to
reinforce information provided by other teammates

the most important member of the patient care team is the
technician

the most important monitor in the dialysis setting is
teammate

which learning domain is behavioral based
affective

which learning domain is knowledge based
cognitive

Mr. Roberts verbalizes how to hand crank his blood pump in the event of an emergency. this is an example of which adult principle?
cognitive

a 58-year old patient who is blind comes to treatment accompanied by his wife. which of these approaches should the dialysis technician use when interacting with the patient?
describe the activities in the immediate environment

according to the new conditions for coverage, all dialysis technicians are to be certified by a national certifying agency or state specific certification by
04/15/2010

the minimum amount of experience required to take the NNCC certifying exam is
6 months

a reason why a dialysis technician is to be certified is
its a new federal regulation

which of the following terms indicates a solution that has the same concentration of solutes as blood?
isotonic

a patient’s potassium levels has been running high between dialysis treatment, so the patient has been working with the renal dietician to restrict potassium in the diet. which of the following fruits should the patient avoid?
bananas

during osmosis, which of the following movements takes place?
fluid moves from an area of lower concentration to higher.

a patient’s granddaughter calls the dialysis center and asks for a progress report on her father, who is a patient in the center. the technician should.
state that patient information cannot be divulged

if a hematoma forms at the access site where a needle infiltrated, the usual intervention is to apply
cold compresses

which of the following actions by a patient increases the risk of blood clots in a venous access?
wearing a top with very tight- fitting long sleeves

a patient is very anxious during an initial hemodialysis treatment and requires extra attention. when the treatment is completed, the patient offers a $20 tip. the technician should
politely decline the tip

which of the following statements by a hemodialysis patient suggests that the patient needs more education?
i should stay on a low protein diet.

the inner part of the kidney is the
medulla

a nephron is comprised of a glomerulus and a
tubule

how much urine does a patient with healthy kidneys usually excrete in 24 hours?
1000 to 2000mL

when removing soiled gloves, the first glove removed should be.
grasped in the opposite gloved hand

which of the following hormones is produced by the kidneys?
erythropoietin

at which stages of chronic kidney disease should a patient being dialysis?
5

which of the following is a genetic disorder that can lead to kidney failure?
polycystic kidney disease

which of the following complains by a patient may be an indication of uremia?
itching

a patient with a right-sided radiocephalic A fistula (above the wrist) needs to have blood test in the lab for a number of different lab test. the blood should be drawn from the
left hand

when setting up hemodialysis equipment, the four thing that need to be checked are [ 1 ] dialysate, [ 2] extracorporeal circuit, [ 3] dialyzer and, [ 4]
machine alarm

generally, the optimal dialysate flow rate for hemodialysis should be
1.5 to 2 times the blood flow rate

a patient is receiving hemodialysis with a dialyzer with an ultrafiltration coefficient [ kuf ] of 10 and a transmembrane pressure [TMP] of 100 mm hg. how much fluid should the patient lose per hour of treatment
1000 mL

under what circumstances can needle be reused for hemodialysis?
under no circumstances

if a emergency ( such as a tornado ) occurs and patients need to be evacuated, which group of patient should be disconnected from dialysis machine first?
patients able to ambulate independently

for patients on hemodialysis a, 1 kg (2.2 lb.) increased in weight in 24 hours is approximately equivalent to fluid retention of
1.0 L.

a patient has been advised to avoid foods high in phosphorus. food that the patient should be advised to limit include.
dairy products

a patient is scheduled for serum creatinine test and asks the technician about preparation the day before the test. the patient should be advised to.
avoid excessive exercise

which of the following ethnic group is most at risk for development of kidney failure?
african american

a patient has buttonhole tracts for access. after the access is cleaned and prepped for treatment, what is the next step?
use a crab picker/ aseptic tweezer to remove the scabs

with buttonhole tracts, the technician should apply pressure as the needle are removed and then for
5 to 10 minutes

if a buttonhole access frequently has long clots that are very difficult to remove, the most likely reason is
failure to use 2-finger hold for needle removal

a patient who had an AV fistula in the forearm developed an aneurysm and has had to have a new AV fistula created in the upper arm. the patient has a temporary catheter in place for dialysis until the fistula has matured. which of the following exercise may help strengthen the AV fistula?
bicep curl

with buttonhole access sites, what should the technician do to prevent ” hubbing ” ?
leave 1/16 th to 1/8 inch of the needle exposed

if a patient has a dialyzer clearance rate of 250 mL/ min with 4- hour treatment, the total volume of blood cleared is
60 L.

what is the most important factor in preventing exsanguination from dialysis line separation?
access site visible

if a patient states that she has been skipping lunch because she is too tired to eat after dialysis, the best solution is to.
report to the nurse and renal dietician

if a patient exhibits signs and symptoms of hemolysis during dialysis, the immediate action should be to
stops the blood pump and clamp the blood lines

the amount of dialysis that a hemodialysis patient is prescribed is based on the removal of
urea

maturation of a prosthetic arteriovenous graft usually takes
3 to 6 weeks

the purpose of the negative germicide test is to ensure that
the reprocessed dialyzer is free of germicides

if a patient develops angina (chest pain) radiating to the neck, jaw, and left arm and the patient’s blood pressure drops during treatment, the technician should notify the nurse and
decreased blood flow rate and ultrafiltration

the technician is reinforcing a patient’s training regarding management of fluid intake. the patient, who still urinates, has a base of 1000 ml intake per day. if the patient urinates 500 ml in a 24-hour period , how much fluid is the patient allowed the following day?
1500ml

the greatest risk of bacteremia (infection in the blood) is associated with which type of vascular access?
dialysis catheter

the external surface of the hemodialysis machine should be cleaned and disinfected at least.
after every patient

when checking the water temperature in the system, the technician records the temperature at 78 F (25.5 C). in order for the reverse osmosis (RO) equipment that is partof the water treatment system to work properly, the water temperature must be maintained at
77′ F to 82’F ( 25′ C to 28’C )

when testing the chlorine levels in the water system, the water sample should be taken from the.
first carbon tank

when considering the chain of infection, the three most common reservoirs of interest include humans, environment and
animals

a solution is a mixture of
solute and solvent

when reinforcing education about weight gain, a patient should be advised that the usual goal for interdialytic weight gain is less than
1.0 kg/d

if a high-pressure alarm for arterial pressure (pre-pum ) sounds during hemodialysis, this could indicate.
drop in speed of blood pump

if a low- pressure alarm for venous sounds during hemodialysis, this could indicate.
clotting in the access

according to KDOQI guidelines, when administering hemodialysis to a patient a, facemark should be worn
for all access connection

the most common complication associated with poor needle site rotation in a graft is
pseudoaneurysm

in hemodialysis, ultrafiltration refers to extraction of
fluid

the patient has develop a small aneurysm and asks the technician to cannulate the aneurysm for the hemodialysis treatment because another patient told this patient that it would be less painful that cannulation of the fistula. the best response is to
advise the patient that cannulating an aneurysm may result i rupture.

a patient experience a cardiac arrest with no pulse or respiration during hemodialysis. after the technician calls for help, the next action should be to
stop dialysis, return blood, and flush access lines with normal saline.

when auscultating a patient’s AV fistula to listen for the bruit, if the technician notes that the bruit is very high pitched, this may indicate
stenosis

when using the buttonhole technique for vascular access for hemodialysis, the needles are placed in.
the same site in a fistula

when inserting a needle for hemodialysis, which of the following increases the risk of infiltration?
rotating the needle 180 degrees

the legal document that assigns a heal care proxy to make decisions in the event that a person is unable to do so is called
durable power of attorney

carbon filters in the water system are necessary to remove
chlorine, chloramine, and organic material

backwashing to free residue from sediment filters in the water system should be done at least
once daily

with the formula for urea kinetic modeling (UKM), the K in the Kt/V formula stands for
urea clearance (mL/ min) plus residual urinary output

when inserting needles into a graft for hemodialysis, the needle tips be should be at least how far apart?
2.0 inches

if a patient is undergoing hemodialysis and the technician notes that a bloodlines has separated and blood has pooled beneath the access site, the first intervention should be to
stop the blood pump

during hemodialysis, the technician would expect a patient’s temperature to rise by about.
0.5 C

approximately what percentage of the total blood volume circulate in the veins?
65% to 80%

patient should be advised to avoid eating during hemodialysis because ingestion of food may result in
hypotension

prior to using a reprocessed dialyzer, a recirculating rinse with NS should be completed with recirculating flow rate through the blood compartment and the dialysate compartment of at least.
200 mL/min for BFR and 200 mL/ min for DFR.

if a hemodialysis patient test positive for hepatitis C virus (HCV), which of the following interventions does the KDIGO guidelines recommend when proving hemodialysis treatment?
adherence to strict infection-control procedures

if a dialyzer is to be reprocessed in 3 hours, the dialyzer must be
refrigerated

when documenting observations about a patient, which of the following is the most appropriate description?
patient is sighing and rubbing hands together

which of the following is the most common cause of hypotension developing during hemodialysis?
removing excessive volume of fluid

which of the following interventions is most likely to decrease hypotension that occurs during hemodialysis?
decreasing the ultrafiltration rate

if an air sector alarm sounds during hemodialysis, the technician should initially
stop the blood pump and clamp the patients venous line

if outflow stenosis of an AV fistula occurs, the thrill usually
is loud and higher pitched and then discontinuous

when reviewing a patient’s food dairy, the technician advise the patient that which of the following protein sources is of low biological value?
dried bean

which of the following vitamins may be removed by hemodialysis
vitamin B

the renal nutritionist has advised the patient to have a diet high in fiber, but the patient is unsure which foods to choose. which of the following foods per serving is highest in insulate fibers?
kidney beans

the primary purification process of the dialysis water is
filtering with activated carbon

if the dialysis center uses hemoclips on dialysis tubing to prevent disconnection, what other precautions should be utilized?
visible access sites/line connections and documentation of integrity every 30 minutes

if during hemodialysis, blood is evident in the used dialysate, this probably indicates
tear in the membrane

if a patient asks a question to which the technician does not know the answer, an appropriate response is
I ‘ll find out for you ”

during routine hemodialysis, a patient’s blood pressure should monitored every
30 to 60 minutes

if a patient develops painful muscle cramps in the hands, feet, and abdomen shortly after hemodialysis begins, the most likely interventions is
saline bolus and/or decreased ultrafiltration rate.

if a hemodialysis patient’s temperature per tympanic membrane thermometer is 37C in order to reduce incidendence of intradialytic ( during dialysis ) hypotension, the temperature of the dialysate solution should ideally be set at
36.5 C

foam in the venous bloodline of a dialyzer may indicate
air embolism

prior to initiating hemodialysis, the ph of the dialyste usually ranges from
7.0 to 7.4

the number one cause of kidney failure in the US is
diabetes mellitus, type 2

the use of topical anesthetic, such as EMLA, to reduce discomfort during cannulation is contraindicated with
buttonhole sites

a patient who has been very alert and shown no sign of cognitive impairment seems confused during dialysis and repeatedly asks the same question. the best initial response is to
notify the nurse

when determining if a new AV fistula is maturing the three factors to assess by palpation are the
thrill, vessel growth, and vessel firmness

which of the following antiseptics used for skin pre for a fistula site has the broadest spectrum antibacterial activity?
2% chlorhexidine gluconate

while a patient is undergoing hemodialysis, chloramine testing of the water system should be conducted every
4 hours

if a patient is to undergo heparin-free dialysis, the optimal blood flow rate is
300 to 400 ml/min

before cannulating An AV fistula for a hemodialysis treatment, the technician examines the patient’s access arm and finds that the access arm appears slightly edamatous, and the skin is pale. on further examination, the technician notes that there are small purple veins evident on the chest wall near where the arm meets the body. the technician should suspect
stenosis

in order to increase survival rates. the ultrafiltration rate for hemodialysis patients should be maintained at least than
12 ml/kg/m

as part of fistula assessment before cannulation, the technician evaluates the thrill and then applies acclusion by placing a finger across the body of the fistula. while the fistula is acceded, the technician palpates and feels both a thrill and a pulse. this probably signifies a (n)
accessory pathway ( collateral circulation )

how much additional proteins should a patient on hemodialysis ingest every day in comparison to a health person
50%

weight gain between hemodialysis treatments should not exceed
5% of dry weight

the technician has calculate the target weight loss for a patent’s hemodialysis session, but the patient insists that the technician has made an error and the target is 1 kg too high. the technician should
recalculate the target weight loss.

the most important reason for placing hemodialysis needles in antegrade position ( in direction of blood flow) is because
it causes less scarring

when preparing dialysate with a 45x concentrate, if the proportioning ratio contains 1.0 part acid and 1.72 parts bicarbonate, how many parts of water are needed?
42.28

if conductivity monitoring stops dialysate from flowing to the dialyzer but instead sends it to the drain (bypass), this means that
the conductivity monitor is malfunctioning

when using a portable digital dialysate meter, such as the D6 (myron L meter ) , to verify that inline meters are accurate regarding conductivity and ph, the technician must
obtain an unused dialysate sample

the target ultrafiltration goal set for the patient is based on the patient’s
dry weight

a patient is to receive a bolus of ulfrationated heparin (25 IU/kg ) at the onset of dialysis and 800 U at one-hour intervals during treatment, which is scheduled from 1 to 5 PM. the first dose is given at 1 Pm and the last dose at
4 pm

when checking the pump occlusion for the blood pump, the technician nots, that the rollers appear to loose.this may result in
cracking of the blood pump segment of tubing

in the hemodialysis machine, a separate air detector is usually placed
between the venous pressure gauge and the venous access

at the completion of dialysis treatment, it is important to rinse back as much blood as possible in order to
reduce the severity anemia

if a power outage occurs and backup power fails, before using the hand crank to return the patient’s blood, the technician must
remove the venous line for the air detector

if a patient’s blood pressure is 142/88 before dialysis and 108/72 at the completion of dialysis, the most likely intervention is
infusion of normal saline to increase blood pressure

prior to opening a sterile packing, the technician spills sterile normal saline on the package. the technician should
discard the package

when washing the hands, the hands should be wet and soap applied and then the hands rubbed together for at least
15 seconds

how long can hepatitis B virus live on surfaces if they are not properly disinfected?
7 days

in the event that a fire occurs in a dialysis center, the RACE method of response includes
rescue, active alarm, contain/confine fire, and extinguish/evacuate

if the technician needs to move boxes of supplies from the floor to elevated shelves, the technician should
bend down at the knees and keep the back straight while lifting

for documentation in the medical record, which of the following includes a correct
0.5 kg

for a hemodialysis cannulation with a blood flow rate of fewer than 300mL/ min, which of the following needle gauge sizes is usually recommended?
17

if the nephrologist has ordered a pre-dialytic and post-dialytic BUN ( blood urea nitrogen) for a patient with an AV fistula, the pre-dialytic blood sample should be obtained from the.
arterial needle

the primary focus of the life safety code is on
protection of people

the dialysis prescription is for a 3- hour treatment with 1.5 kg to be removed. during treatment 250 mL is used as saline prime and 200 mL as saline rinseback. the patient ingested 150 mL of fluid that contains oral proteins supplement. the ultrafiltration rate to be removed per hour is
700 ml

if the ultrafiltration rate to be removed per hour is 600 ml and the dialyzer kuf is 40 what is the transmembrane pressure (TMP)?
15 mm hg

if using a combination chlorhexidine gluconate and alcohol skin prep (such as chloraprep before cannulation, how much skin contact time is required?
30 seconds

which of the following preventive measures is usually the best approach for patients who routinely experience hypotension during dialysis because of volume-related problems associated with large weight gain between treatments?
increasing restrictions of sodium intake

if a patient undergoing hemodialysis and in the care of the technician has a persistent cough, standards precautions require that the
patient wear a mask

what color is arterial blood tubing for hemodialysis most often color-coded?
red

during hemodialysis, how much blood is usually outside of a patient’s body at one time?
100 to 250mL

during the first week of treatment with a new AV fistula, the initial needle size and blood flow rate are usually ~
17-gauge needle and blood flow of 200 to 250 mL/min.

with an AV fistula, cannulation should usually be done at an angle of
25-35

a hemodialysis system includes blood volume monitoring to assess changes in the patients democrat.if the hematocrit increases rapidly, this means
blood volume is decreased

a patient has a newly created AV fistula. when assessing the fistula, the technician raises the patients involved arm above the head. in this position, the fistula should
collapse

during cannulation, a tourniquet should be used
while placing needles in all AV fistula

the difference between a buttonhole needle and a standard needle for hemodialysis is that the buttonhole needle has a
blunt tip

if a hemodialysis patient has an extreme fear of needle and the physician describe EMLA cream to prevent pain, the cream must be applied
60 minutes prior to treatment

following formation of an AV fistula and beginning of hemodialysis, the technician notes that the patient’s nailed and skin on the hand below the fistula are cyanotic during hemodialysis, and the patient complains of pain in the hand. this is likely an indication of
steal syndrome

after a hemodialysis needle is removed at the end of treatment, the correct procedure is to apply pressure to
each access site with two fingers for up to 20 minutes

a patient who wants to learn self-cannulation may do so if
the patient receives appropriate training

thrombosis is newly-created access may be caused by
low blood pressure

if a new patient has Hero (hemodialysis reliable outflow) graft, the cannulation site is generally in the
proximal upper arm (below the axilla).

if effluent (used dialysate ) spills into the floor, the technician should
notify staff trained in hazardous waste removal

a patient is recovering hemodialysis with 17-gauge needle, but his blood flow rate has been increased to 450ml/min. the patient complains of symptoms consisten with hemodialysis ( breakdown of red blood cells) sudden onset of chest tightness, back pain, dyspnea, reddening ok skin color, and blood in the venous line appearing port-wine in color. what is likely cause of this complication?
needle too small for blood flow rate

when handing off a patient to another staff person, the technician should always
follow the hand-off protocol established by the institution

the technician should ensure when exposing the opening of a central catheter to air (such as when removing the cap or a syringe) that the
clamp is closed

the hemodialysis center has instituted a “zero lift” policy. the primary purpose of such a policy is to
prevent injuries

if the dialysis center has an extra hemodialysis machine that has not been used in the previous 2 weeks, how frequently does this machine need to be disinfected?
every 48 hours

five minutes after a patient begins a hemodialysis treatment, the patient has difficulty breathing and rapidly develops generalized hives, itching and swelling about the eyes. after asking the nurse for help, the next action should be to.
stop the dialysis and clamps all the lines

a patient routinely experiences hypotensive episodes near the end of a session and experiences malaise (tiredness), muscle cramps, and dizziness after dialysis. what is the most likely cause?
the dry weight is set too high

an adolescent patient has required frequent hospitalization because of nonadherece to be the treatment plan. the best approach to take with the patient is to
ask the patient how the staff can help the patient manage better.

when the first carbon tank ( the worker) in the water system is exhausted, the next step is to
remove the first tank, move the second tank to first position, and add a new tank in second position.

following dialysis, the patient’s blood pressure in semi-reclining position is 128/86. the technician takes a series of blood pressure over 3 minutes after the patient stands. at which blood pressure should the technician alert the nurse that the patient is exhibiting orthotic hypotension?
108/74

the technician is able to feel no pulse or thrill along the patient’s outflow vein and can detect no bruit. the technician should suspect
thrombosis

if a dialysis patient is very large in both stature and weight, which of the following is most likely to improve the effectiveness of hemodialysis
increase dialyzer size

the duties that the technician is allowed to carry out depend primarily on the __
state regulations/standards of practice

if a dialysis center carries out dialyzer reprocessing and uses peracetic acid as the germicide, the contact time needed is _
11 hours

if a dialysis center uses electronic charting, the technician may share his/ her password with__________
no one

when creating a buttonhole tract, how many consecutive cannulations are usually required before the tract is adequately formed?
8 to 10

if a patient complains of persistent itching, which of the following nonprescription methods may help relieve symptoms?
take oatmeal baths

when testing water for the level of chloramine, the technician finds that the total chlorine levels is 1.1 parts per million (ppm) and the free chlorine is 0.9 ppm. therefore, the chloramine levels is
0.2 ppm

patient receiving hemodialysis should be advised to avoid salt substitutes because the substitutes often
contain potassium

patients with uremia develop foamy or bubbly urine because _
proteins leaks into the urine

which of the following is a function of the end-stage Renal Disease Networks?
help resolve patient-staff conflicts

when using a slide board to transfer a patient from the dialysis chair to a wheelchair, at what time angle should the slide board be placed between the two sitting surfaces?
45- degrees

when the technician is teaching a patient to self-cannulate using the tandem-hand technique the
patient places the thumb and finger behind the technician’s thumb and finger.

a patient has a right internal jagular central venous catheter for hemodialysis, but a pressure alarm sounds during treatment indicating that the blood flow rate is lower than prescribed. on checking, the technician finds that the lines appear clear and there are no signs of kinking, bleeding or air entering the system. the next intervention should be to _.
lower the patient’s head and ask the patient to cough.

the PDCA cycle used for continuous quality improvement refers to __.
the plan- do- check -act

substances that form ions (changed particles) are_____.
electrolytes

which members of the dialysis center team are responsible for carrying out the quality assessment and performance improvement program at a dialysis center?
all staff

a new female patient is admitted to the dialysis center in the company of a male who appears to be her spouse. if the patient’s chart lists her name a mary Jo Johson, how should the patient be addressed?
ask the patient

infections in buttonholes tracts are almost always caused by __
improper scab removal

what percentage of the water that is filtered out of the blood by the glomeruli is excreted as urine?
1%

which of the following hormones directs the bone to produce more red blood cells?
erythropoietin

to reinforce teaching, the technician should remind patients with kidney failure that they should avoid.
smoking

which of the following is a genetic (inherited ) disease that can lead to kidney failure?
polycystic kidney disease

with uremia, as waste products build up in the blood, this can result in
ammonia breath

the aluminum level in dialysis water should be _.
< 0.01 mg/L

normal saline has a sodium,m chloride concentration of
0.9 %

as part of anemia management for patients receiving hemodialysis, the technician should_________.
rinse back as much blood as possible

patients receiving hemodialysis are at risk for amyloidosis. which of the following symptoms may indicate amyloidosis?
joint pain

a patient who experiences numbness, tremors, muscle spasms, and muscle pain may have low levels of ___-.
calcium

which of the following laboratory tests is often done before and after hemodialysis to measure the effectiveness the dialysis in removing waste products from the body?
blood urea nitrogen

a patient beginning hemodialysis usually has a hemoglobin test every __.
one to two weeks

the most common treatment for patients with end-stage kidney disease in the united states is_______
daytime in-center hemodialysis.

which of the following is an indication that a patient may be losing fat and/ or muscle mass even through he or she reaches the target dry weight after treatment?
patient become short of breath if lying flat after treatment

which member of the hemodialysis care team is responsible for setting up the plan of care for a patient?
nephrology

if a patient coms for hemodialysis treatment but weighs less than the target dry weight before treatment, the technician should___________
notify the nurse and dietitian

the first step in treating malnutrition in a dialysis patients is usually to_________
encourage the patient to eat more.

one-half teaspoon of salt is equal to about how many milligrams of sodium?
1000mg.

because of increasing phosphorus levels, a patient has been prescribed a phosphate binder, but the patient is unsure when to take the medication. the best advice is to take the medication____________.
with meals

if patient is concerned about loss of income because of the need for hemodialysis, the best person to speak to the patient about concern is the.
social worker

when using active listening, which of the following is a good example of sort, ope-end question to use to encourage patient communication?
what does the pain in your abdomen feel like?

if a patient has refused to participate in eduction, ask no questions, and shows no interest in learning about hemodialysis, the best approach for the technician is to __.
talk through all the steps in the procedures

diffusion is a process that involves __
movement of solutes though a semipermeable membrane from an area of higher

a dialysis machine is set up to use 36.1x parts dialysate. if the concentrate proportioning ratio calls for 1.00 part acid and 1.10 parts bicarbonate, how many parts of water are needed?
36.1

about what percentage of body water is in the intravascular space (inside blood vessels)?
10%

the mass transfer coefficient of a dialyzer refers to _
how well solutes pass through the dialyzer membrane

when using an alcohol-based hand rub, the hand rub should be applied to one palm and the hands rubbed together________.
until dry

the three process that affect dialyzer clearance are (1) diffusion, (2) convection, and (3)__.
adsorption

if a dialyzer has a sieving coefficient of 0.6 for a solute, what percentage of the solute should pass through the membrane?
60%

most dialyzer membrane used today are made of
synthetic

if a dialyzer has urea clearance rate of 200 ml/min and a blood flow rate of 400 ml/ min, what percentage of the 400 ml of blood is cleared of urea in one minute?
50%

the storage time for a reprocessed dialyzer is __
set by the manufacturer

if a low-conductivity alarm sounds, the technician should suspect-
lack of concentration in the proportioning system.

if the blood leak alarm sounds during a hemodialysis treatment and the blood pump stops and lines clamp but the dialysate appears clear, the technician should_____________.
use hemastix to check the extent of the leak

when using a portable ph monitor to verify that an in-line monitor reading is correct, the technician should first______________.
test a solution with a known ph

if a solution has an equal number of acid and base ions is neutral, the ph is __.
7.0

a volumetric ultrafiltration control system has _.
two identical dialysate chamber

the extracorporeal circuit carries blood
from arterial access to dialyzer and back to the venous access

a drip chamber that contains a fine mesh screen should be placed _.
between the the dialyzer and the venous access

if a low-pressure alarm for arterial pressure
(prep ump) sounds during dialysis, this could indicate
infiltration of the arterial needle

if a high-pressure alarm for the predialyzer
(post pump ) sounds during dialysis, this could indicate____________.
clotted dialyzer

the lifespan of an arteriovenous graft is usually _.
3 to 5 years

the heart must work harder when patients have an arteriovenous fistula because the
blood flow is faster

the technician should alert the nurse that the patient is experiencing tachycardia (rapid pulse) when the pulse exceeds
100

a patient has opted to have a buttonhole tracts for hemodialysis. which of the following the most important factor for creation of a buttonhole tract?
the same cannulator should do treatments until the tunnel is established

when a patient is utilizing home hemodialysis, the purpose of teaching the patient to “snap and tap” the tubing and filter is to ___.
remove air bubbles

an 85-year-old patient with end-stage kidney disease and multiple health problems has started hemodialysis, but the patient is now refusing treatment, stating that death is preference to the loss of independence and continued illness. the technician should _.
respect the patient’s decision

after a patient receives the initial heparin bolus for hemodialysis, when should dialysis be initiated?
within three to five minutes.

which of the following signs and symptoms are indications of infection of an arteriovenous fistula or graft?
redness, tenderness, and swelling

if the patients feels unsure about cannulation a patient’s fistula because or irregular vessel shapes, the best solution is to__________.
ask a more experience staff member to cannulate

which of the flowing needle sizes has the largest lumen (diameter )?
14 gauge

retrograde (against the blood flow) insertion of arterial dialysis needle can result in______________.
increasing scarring

the best method to secure a dialysis needle is probably __.
butterfly technique.

when removing an access needle at the complication of hemodialysis, the technician completely removes the needle before applying pressure. this technique __.
is the incorrect procedure

if a patient is terrified of needle and often feels faint and nauseated during cannulation, the best position to place the patient in during cannulation is __.
flat

if the blood in the arterial line looks very dark( black blood syndrome ”), the likely cause is
recirculation

the best method to prevent aneurysm in a fistula is to______.
use proper technique when inserting needles

if air detector alarm sounds, the blood pump stop, and the venous line clamp, but the technician notes air in the venous line and is concerned that some air may have entered the patient’s vein, the technician should immediately_____________.
place the patient in the trendelenburg position (head below heart ) on the left side.

the heparin infusion line should generally be place
in the arterial line between the blood pump and the dialyzer.

if the power goes out during a treatment, the blood in the bloodlines is __.
returned by hand cranking the blood pump

if a technician notes a high-pitched bruit, a ”water-hammer” (pounding ) pulse, and evidence of clotting in the extracorporeal circuit during hemodialysis, the technician should suspect__________.
stenosis

if the skin above an arteriovenous graft appears taut and shiny, and previous needle sites are unhealed, the technician should alert the nurse to possible__________
pseudoaneurysm

if the rate of blood flow is low, this increases the risk of__________.

dialysate is made by mixing water with acid and bicarbonate concentrates. Acid concentrate contains __.

when a patient’s arteriovenous graft has healed and is ready for the first cannulation, what needle size is usually indicated?

when placing needles in an arteriovenous graft, how far away from the previous needle sites should cannulation be done?

the most common site for development of stenosis in an arteriovenous graft is at __.

which of the following sites for a central venous catheter may prevent future venous access in the limb on the same side?

the preferred solution to clean exit sites for central venous catheter is _

what length of needle is usually needed for a forearm arteriovenous (AV) fistula?
0.6 inch ( 3/5 inch).

when drawing blood sample from the injection port of the arterial bloodline, the personal protective equipment that the technician should wear is _.
gloves, facemark, eye protection, and gown

the first step in teaching a patient to self-cannulate is to tell the patient about ___.
how the patient’s graft/ fistula woks

when using “touch cannulation” to cannulate a buttonhole tract the cannulator. __.
holds the tubing 1 to 2 cm behind the needle

when you taking a manual blood pressure reading on a patient’s nonaccess arm, the blood pressure should be checked._
at the heart level

if an item, such as a piece of equipment, s considered ” clean “, this means it is _.
disinfected but not free from all germs

when hanging a new bag of normal saline, the technician touches the spike to the outside of the of the bag. the technician must____________.
discard the spike and attached tubbing

how frequently should patients and staff members of a dialysis center have tuberculosis skin tests?
once a year

if a patient has completed a dialysis session and is walking toward the door with the technician but becomes unsteady and beings to fall, the first thing the technician should do is to _.
ease the patient to the floor

the first thing to do when using a portable lift device, such as hayer lift, to transfer an obese patient is to ___.
check the lift’s weight limit and sling size.

a small fire begins in water receptacle, and the technician uses a fire extinguisher to put out the flames. if using the P.A.S.S. protocol, the technician should____________.
pull the pin, aim the nozzle, squeeze the handle, and spray from side to side

when using reprocessed dialyzerr for treatment, the first thing that the technician should check the dialyzer for is the _.
remove air and germicide

when setting up hemodialysis equipment with a reprocessed dialyzer before a treatment, the primary purpose of priming the bloodline and dialyzer with normal saline is tp ___.
patient’s name

which of the following signs or symptoms may indicate that a patient is below dry weight following a hemodialysis treatment?
low blood pressure

a radial pulse is accessed at the
wrist (thumb side)

in modern dialysis machines, in order to start a treatment, the technician must enter into the machine the __.
total number of milliliters to be withdrawn and the duration of treatment

if a patient develops hypotension (low blood pressure) during a hemodialysis treatment, and the technician finds that the patient has developed rapid and irregular heartbeat, then the technician should notify the nurse and____________.
decrease the ultrafiltration rate

a patient is nearing the end of the treatment and complains of headaches and nausea and appears very restless and anxious. the technician checks the vital signs and finds that the blood pressure is 180/100, and the pulse is 72 and is slightly irregular. the technician should suspect__________.
disequilibrium syndrome

then permeability of a dialyzer membrane to wear is indicated by its __.
ultrafiltration coefficient

if a patient requires a blood draw for laboratory test before a hemodialysis treatment, and the laboratory has provided four vacuum tubes for the blood sample, then the technician should ___.
check the order of the draw

if a patient is not to receive a saline prime at the beginning of the hemodialysis treatment, and the saline prime is drained into a waste container on the side of the dialysis machine, then the technician must connect the venous line to the access when blood reaches the __.
venous chamber

if a patient develops a nosebleed during a hemodialysis treatment, this could indicate that the______.
heparin dosage is too high

a patient undergoing hemodialysis for the first time does well initially, but at 15 minutes into the treatment, she begins to complain of back pain, itching, slight shortness of breath, and nausea. the most likely cause of these signs and symptoms is _.
first-use syndrome

the best method to prevent leg cramps during hemodialysis is to
correctly calculate the target ultrafiltration goal

after a blood sample is collected into a blood tube, the technician should______________
gently invert the tube back and forth about eight times

if the dialysate is too warm, it can result in________
hemodialysis

after ringing and cleaning a dialyzer that is to be reprocessed, the performance test or tests that must be performed are____________.
total cell volume test and test leak test

if the water pressure downstream of a multimedia filter in the water system is decreased, this indicates a need to __
backwash te filter

if a dialyzer is to be reprocessed at the completion of a dialysis treatment, the dialyzer should be __
flushed with normal saline

if the water distribution system uses an indirect feed system, the speed of the water flow through the distribution system should be _______
3 feet per second

how often should colony counts and limulus amebocyte lysate tests be conducted on the water feeding the bicarbonate mixer?
monthly

heparin B is spread through _.
contact with blood or body fluids

if testing a water sample for bacteria, the sample should be reprocessed at room temperature within.
1 to 2 hours

if there patients having dialysis at the same time in a center that uses a central dialysate delivery system complain of sudden onset of nausea, the technician should suspect a problem with the _.
water system

one fluid ounce is equal to __.
30 ml.

a patient and his and his spouse are undergoing training for in-home hemodialysis, but the patient tells the technician, ” I ‘ m so nervous. what happens if I can’t learn to do this? the best response is __.
you will only do in-home dialysis when you feel ready.”

in the estrecorporeal circuit, the highest positive pressure is found at the
arterial header of the dialyzer.

if testing before a hemodialysis treatment shows that the patient’s blood level of potassium is too low, the level of potassium in the dialysate is usually __
increased

the factor that has the most influence on a dialysis patients’s level of thirst between treatments is _.
salt intake

the purpose of using a water softener in the water distribution system is to ___
reduce levels of calcium and magnesium

patients receiving high-flux dialysis may have _.
shorter treatment times.

if a patient becomes nauseated and vomits during hemodialysis, the initial intervention should _.
assesss for hypotension

a HERO graft is indicated for __.
patient with stenosis/blockage of central veins leading to the heart.

water is removed from a patient’s blood during hemodialysis by ___-.
osmosis

the fluid velocity (speed ) of blood through tubbing is based on flow rate and
tube diameter

when a dialysis treatment starts, the first point of restriction is the __.
needle

if there is a kink in the venous bloodline between the venous pressure gauge and the venous access, what type of alarm should sound?
high venous pressure alarm

in an arteriovenous fistula, the venous portion enlarges because of the ___.
high-pressure flow of blood from an artery

if an arterial venous fistula, is referred to as autogenous, this means that fistula _.
is made from the person’s own blood vessels

brachiocephalic arteriovenous fistula is located in the __.
upper arm

if using ethyl chloride spray on an access site to reduce discomfort, __
the patient should not feel the needle at all

signs and symptoms of steal syndrome indicate _.
the hand on side of access feel cold and painful

if the technician is unable to feel a pulse or thrill or to hear a bruit in the outflow vein of an arteriovenous fistula, this may indicate___________.
thrombosis

when preparing to administer hemodialysis through a central venous catheter, the technician is unable to flush the catheter with normal saline because of resistance. the technician should. __.
hold the treatment and notify the nurse

which of the following kinds of dialysis needles can be used without a safety device to prevent needle stick injuries?
buttonhole needles

which of the following is an extracorporeal alarm rather than a dialysate alarm?
air detector

a normal respiration rate for most adults is __.
12 to 16 per minutes

when a patient is undergoing hemodialysis, all technical systems should be monitored at least__________
every 30 minutes

after the onset of hemodialysis, an anaphylactic reaction usually occurs within __.
5 to 10 minutes

if a reprocessed dialyzer has been accidentally exposed to two germicides, the dialyzer _.
must be discard

the most common site for placement of a straight graft is the __.
forearm

which of the following is in indication of a pyrogenic reaction to hemodialysis?
development of chills and fever 45 minutes into treatment

if a high-pressure alarm for venous pressure sounds during hemodialysis, this could indicate_______________.
infiltration of the venous needle

Leave a Comment

Scroll to Top